LSAT and Law School Admissions Forum

Get expert LSAT preparation and law school admissions advice from PowerScore Test Preparation.

User avatar
 Dave Killoran
PowerScore Staff
  • PowerScore Staff
  • Posts: 5852
  • Joined: Mar 25, 2011
|
#82504
Complete Question Explanation
(The complete setup for this game can be found here: https://forum.powerscore.com/lsat/viewtopic.php?t=14240)

The correct answer choice is (D)

The question asks for a viable list of tennis players, in the correct ranking order. From the first rule, O plays tennis, and so answer choice (E), which does not contain O, can be eliminated.

From the fifth rule, when M plays tennis, O must rank higher than S, who must rank higher than M (O > S > M). This information eliminates answer choice (C), as the players violate the ranking order.

From the last rule, when P plays tennis, K must play tennis. This eliminates answer choice (A), which does not include K. Further, from the same rule, when P plays tennis, K must rank higher than O, who must rank higher than P (K > O > P). This information eliminates answer choice (B), as the players violate the ranking order.

Thus, answer choice (D) is proven correct by process of elimination.
User avatar
 lalalala
  • Posts: 13
  • Joined: Aug 04, 2023
|
#103193
Hi, I got this right but I was a little confused when picking the answers. Doesn't D violate the last rule, which makes the order K - O - P? The correct answer's order is O - S - K - M. Isn't K supposed to be ranked higher than O?
 Rachael Wilkenfeld
PowerScore Staff
  • PowerScore Staff
  • Posts: 1358
  • Joined: Dec 15, 2011
|
#103208
Hi lala,

That's a conditional rule. It only applies IF P plays tennis. In answer choice (D), P does not play tennis, so the ordering given does not apply. This game is a bit tricky with the conditional rules. Remember that conditional rules only apply once the sufficient condition occurs; if the sufficient condition does not occur, you can't draw a conclusion.

Hope that helps!
 boehmejayne@gmail.com
  • Posts: 11
  • Joined: Oct 12, 2023
|
#104527
Doing this question by process of elimination is a brilliant idea. Thanks! I got it right but did it the long way.

Get the most out of your LSAT Prep Plus subscription.

Analyze and track your performance with our Testing and Analytics Package.